LSAT and Law School Admissions Forum

Get expert LSAT preparation and law school admissions advice from PowerScore Test Preparation.

 lsatprep1215
  • Posts: 33
  • Joined: Dec 16, 2019
|
#73572
Hi, I don't know how to do this question because I don't know what is the gap in the argument.
My diagramming is like this: infrequent + not interfere :arrow: Preserve
Frequent :arrow: note with explain
note with explain more than once :arrow: General Statement
Typo in modern work :arrow: without explain

I don't know what to look for in this question, can someone help?
 Jeremy Press
PowerScore Staff
  • PowerScore Staff
  • Posts: 1000
  • Joined: Jun 12, 2017
|
#73589
Hi lsatprep1215,

This is a Must Be True question, because the question stem asks us to identify an answer choice that "follows logically from" the statements in the stimulus. In a Must Be True question, we ordinarily don't expect to find an argument, and that is the case here as well. So, there's no need to focus on argument gaps (since there is no argument).

There are, as you've correctly noticed, a LOT of conditional statements in the stimulus. It's helpful to jot down a brief diagram of each one. Some Must Be True questions also require you to make connections between conditional statements (link them together into a long conditional chain). This is not one of those questions. There are no easy links between the various conditionals. Instead, what we have to do is simply understand what each of the various rules implied by the conditional statements means.

The first statement you've diagrammed correctly, but let's reflect on its meaning. It states a rule that applies to older works, direct quotations in those works, and specifically the archaic spelling and punctuation in those direct quotations. It says that if the spellings/punctuations are infrequent and don't interfere with reader comprehension, then they should be preserved. Notice how answer choice C is, in essence, a Mistaken Negation of that conditional rule: if the spelling DOES interfere, then you should NOT preserve it. A Mistaken Negation must be wrong on a Must Be True question.

The second statement applies to the same categories (older works, direct quotations, archaic spellings/punctuations), and says if the spellings/punctuations occur frequently, then the editor may modernize them (EITHER with a note in the text, OR with a general statement in the preface, depending on how many quotations are modernized). Notice how answer choice D is right on the mark for this rule. It's about an older work, a direct quotation, and an archaic punctuation. It says that if the archaic punctuations occur frequently, the editor may modernize them. Bingo! Notice also how answer choice E contradicts this rule. It says for just one modernization to put the explanation in the preface. That's against this rule, which says for just one modernization, you should put the explanation in the text (not the preface).

The third statement is about different categories (modern works, quotations, typographical errors). It says the errors may be corrected without explaining them. Notice that the problem with answer choice A is that we don't know if it's talking about modern works or older works. So we don't know whether this rule (make the correction with no explanation) or the "older works" rule (make the correction, but include an explanation) applies. Notice also that the problem with answer choice B is that it's talking about modern works, but the only rule we have about modern works is for typographical errors. We don't know what the rule is for archaic spellings in modern works.

There's no real shortcut through this question, unfortunately. It's very much tied to your close reading of each of the rules, and finding an answer choice that conforms to one of those rules. That's why this was question 19 in its section. It's in a harder part of the LR section, where the test makers are trying to slow you down. Strategically, it might be a good bet to do some of the shorter/simpler questions in the section before you tackle this one.

I hope this helps!

Jeremy
 lsatprep1215
  • Posts: 33
  • Joined: Dec 16, 2019
|
#73590
Thanks so much for the explanation! Now I understand why I am having trouble approaching this question, I thought it is an assumption question when I saw "follow logically".

Get the most out of your LSAT Prep Plus subscription.

Analyze and track your performance with our Testing and Analytics Package.